Finding the Antiderivative of f(x) from 1 to 4 - Graph Analysis

Click For Summary
The discussion focuses on finding the antiderivative of the function f(x) from 1 to 4 using graph analysis. The solution involves breaking the area under the graph into two integrals: one representing a negative trapezium and the other a triangle, resulting in a total integral value of -1.5. An alternative approach suggested is to consider the areas from 3 to 3.5 and 3.5 to 4, which cancel each other out, simplifying the integral to just from 1 to 3, yielding -3/2. The average value of the function over the interval is calculated as -1/2. The discussion concludes with a light-hearted acknowledgment of the mathematical process.
t_n_p
Messages
593
Reaction score
0

Homework Statement



http://img296.imageshack.us/img296/7923/img0192re9.jpg


The Attempt at a Solution



Hope you can see what I've written on the top of the page, but basically I want to find the antider. of f(x) from 1 to 4 (from the graph I suppose). So that should be the negative area under the graph plus the little triangle just above the x axis?
 
Last edited by a moderator:
Physics news on Phys.org
Break it up into 2 integrals,
\int^{3.5}_1 f(x) dx + \int_{3.5}^4 f(x) dx

We can see the 1st integral is just the negative of a trapezium, and it is -7/4.
The 2nd integral is just a triangle, area 1/4. So the integral evaluates to -1.5.

A better way to have done it would have been to see the area from 3 to 3.5 and 3.5 to 4 cancel out making the integral \int_1^3 f(x) dx, which is -3/2 as expected. Turns out you don't see an anti derivative, but if you were really really desperate, you could have written a piece wise definition of f(x) where f(x) = -x+1 for x between 0 and 2, f(x)=-1 for x between 2 and 3, so on, so forth..and done it the long way.

Anyway, we have the integral now, you can do the rest.
 
So basically average value = (1/3)*(-3/2)
= -1/2
 
Exactly. Good Work.
 
Gib Z said:
Exactly. Good Work.

*bows down to maths god*
 
Question: A clock's minute hand has length 4 and its hour hand has length 3. What is the distance between the tips at the moment when it is increasing most rapidly?(Putnam Exam Question) Answer: Making assumption that both the hands moves at constant angular velocities, the answer is ## \sqrt{7} .## But don't you think this assumption is somewhat doubtful and wrong?

Similar threads

  • · Replies 15 ·
Replies
15
Views
2K
Replies
3
Views
2K
Replies
32
Views
3K
  • · Replies 9 ·
Replies
9
Views
3K
  • · Replies 1 ·
Replies
1
Views
2K
  • · Replies 5 ·
Replies
5
Views
2K
  • · Replies 3 ·
Replies
3
Views
2K
Replies
2
Views
2K
  • · Replies 7 ·
Replies
7
Views
2K
Replies
4
Views
2K